You are on page 1of 10

BBA+IPM

QA : BBA + IPM Specific 01


Number System

Number of Questions : 25 CEX–5670/20

Class Exercise 5. How many natural numbers are there between


200 and 400 which are divisible by
1. There is a two-digit number, which is equal to I. both 4 and 5 ?
the sum of the squares of its digits. What is II. 4 or 5, or 8 or 10 ?
the sum of the digits of that number?
(a) 7 (a) 9, 79 (b) 10, 80
(b) 5 (c) 9, 81 (d) 10, 81
(c) 8
6. A two-digit number ‘ab’ is six times the sum
(d) Such a number is not possible
of its digits. Find the value of ab.
(a) 625 (b) 81
2. Convert 2.52145145145… into a fraction.
(c) 1024 (d) 3025
25398 25893
(a) (b)
9990 9900 7. If a natural number ‘N’ is decreased by the
sum of its digits, then the resulting number
25139 251893 is ‘M’. If ‘M’ is increased by the sum of its
(c) (d)
9900 99900 digits, then the resulting number is ‘N’. Find
the largest such three-digit number N.
3. Let x, y and z be arbitrarily chosen distinct (a) 998 (b) 999
prime numbers, where x< y < z. Which of the (c) 990 (d) 996
following options is necessarily false?
8. How many four-digit numbers ‘pqrs’ exist such
x+y that both ‘pq’ and ‘rs’ are distinct two-digit
(a) ≥1
z perfect squares?
(a) 30 (b) 32
2 y+z
(b) x y + is odd when x > 2 (c) 42 (d) 20
x
9. What is the units digit of 1781 + 2781 + 3781
x3 + 3y
(c) is even + … + 9781?
z (a) 5 (b) 3
(d) None of these (c) 1 (d) 7

4. Which of the following numbers is exactly 10. Find the last two digits of the number
divisible by 99? N = 29933.
(a) 3572403 (b) 135732 (a) 01 (b) 81
(c) 913464 (d) 114345 (c) 89 (d) 99

Number System Page 1


11. What is the digit at the hundreds place of the I. After how much time will all six of them
number N = (101)9? vibrate together?
(a) 9 (b) 1 II. How many times will they vibrate together
(c) 0 (d) 2 in 30 min after starting?
(a) 60 min, 30 times
12. What is the digit at the tens place of the (b) 60 s, 31 times
7 (c) 60 s, 30 times
number N = 611 ? (d) 60 min, 31 times
(a) 3 (b) 1
(c) 5 (d) 9 17. The highest common factor of two numbers
is 5!. Which of the following can possibly be
13. Answer the question on the basis of the the LCM of these two numbers, if one of the
information given below. numbers is 8! ?
In case of some particular dates, when written (a) 9! – 1 × 8! (b) 9! – 2 × 8!
in the format ‘mm/dd/yy’, it is found that (yy) (c) 9! – 3 × 8! (d) 9! – 4 × 8!
= (mm) × (dd).
18. If 2100! = (504)p × q, where q is not a multiple
For example: 22nd April 1988 is written as
of 7, then find the value of p.
04/22/88, where 88 = 4 × 22.
(a) 346 (b) 305
(c) 388 (d) 348
Between the year 1950 and 2000, what were
the maximum possible number of days in a
19. If A is the product of first 100 multiples of 2
single year, in which this could have and B is the product of first 10 multiples of 5,
happened? then find the number of zeros at the end of
(a) 7 (b) 6
(c) 8 (d) 9 A
.
B
14. A is a positive integer such that A is a multiple (a) 16 (b) 18
of 180 and has 40 factors. If A is less than (c) 14 (d) 12

A 20. Two numbers, x and y, are such that when


3000, then the value of is
40 divided by 6, they leave remainders 4 and 5
(a) 240 (b) 60 respectively. Find the remainder when
(c) 270 (d) 54 (x2 + y2) is divided by 6.
(a) 2 (b) 3
15. An,m represents a set of natural numbers less (c) 4 (d) 5
than 10000, whose sum of digits is n and
which are divisible by m. How many elements 21. What is the remainder when 784 is divided by
are there in A33, 6? 2402?
(a) 10 (b) 6 (a) 1 (b) 6
(c) 7 (d) 16 (c) 2401 (d) 2

22. What is the remainder when 7413 – 4113 +


16. Six strings of a violin start vibrating
7513 – 4213 is divided by 66?
simultaneously and they vibrate 3, 4, 5, 6, 10
(a) 2 (b) 64
and 12 times in a minute. Find
(c) 1 (d) 0

Page 2 Number System


23. What is the remainder when 531111 is divided 5. What is the remainder when the product of
by 51? 100 consecutive positive integers is divided
(a) 25 (b) 9 by 99?
(c) 42 (d) 26 (a) 0 (b) 34
(c) 97 (d) 1
24. If (12323)4 → (P)8, then find the value of P.
(a) 653 (b) 673 6. If N = 23 × 32, then how many sets of 2 distinct
(c) 763 (d) 563 factors of N are coprime to each other?
25. What will be the value of (54321 – 12345) × (a) 20 (b) 19
100, converted back to decimal system, if all (c) 18 (d) 17
the numbers mentioned are in base 6?
(a) 522200 (b) 420320 7. Find the number of positive divisors of N2 such
(c) 4153200 (d) 201600 that the positive divisors are less than N and
do not divide N for N = 217 × 39 × 53.
Home Exercise (a)1936 (b)1724
(c)1564 (d)1608
1. If the sum of ‘n’ consecutive integers is 0,
then which of the following statement(s) must 8. A number has exactly 4 factors. How many
be true? such two-digit numbers are possible?
I. ‘n’ is an even number. (a) 40 (b) 39
II. ‘n’ is an odd number. (c) 30 (d) 29
III. The average of the ‘n’ integers is 0.
(a) I only (b) II only 9. If P is a prime number, then find the LCM of P
(c) III only (d) II and III and (P + 1).
(a) P(P + 1) (b) (P + 2)P
2. If ‘bcd’ is a three digit number whose square (c) (P + 1)(P – 1) (d) (P – 1) (P + 2)
ends in ‘abcd’, then what is the value of
a+ b+c+d? 10. Let S be the set of all natural numbers less
(a) 27 than 120 such that the HCF of any element
(b) 13 in S and 120 is 1. Find the number of elements
(c) 16 in S.
(d) Cannot be determined (a) 104 (b) 56
(c) 64 (d) 32
3. Shaurya writes the first hundred whole
numbers. Let, ‘A’ and ‘B’ be the number of
11. How many three-digit numbers exist, which
times he writes ‘0’ and ‘9’ respectively. Find
when divided by 3, 4, 5, 6 and 7 leave
the value of A + B.
remainders 1, 2, 3, 4 and 5 respectively?
(a) 31 (b) 30
(a) 5 (b) 4
(c) 34 (d) 32
(c) 3 (d) 2
4. Let P be the set of even integers ‘a’ such that
12. What is the largest power of 12 that would
125 ≤ a ≤ 275, ’a’ is divisible by 7 but not by
divide 49!?
11. How many elements does ‘P’ contain? (a) 22 (b) 23
(a) 12 (b) 11 (c) 21 (d) 24
(c) 10 (d) 9

Number System Page 3


13. How many consecutive zeros will be there at 18. What is the remainder when 650 is divided by
the end of the product (2!)2! × (4!)4! × (6!)6! × 215?
(8!)8! × (10!)10!? (a) 1 (b) 36
(a) 10! + 6! (b) 2 (10!) (c) 5 (d) 214
(c) 10! + 8! + 6! (d) 6! + 8! + 2(10!)
19. In decimal system there are 10 digits 0, 1,
14. The sum of the digits of a number A is 13. 2, 3, . . . till 9. Similarly, there is a system
which has only 3 digits 0, 1 and 2. Numbers
What is the remainder when A 4 is divided by
represented in such systems are called base
3?
3 or trinery numbers. N = 389 + 1 is a number
(a) 1 (b) 2
in the decimal system. M is the trinery
(c) 0 (d) Either (a) or (b)
equivalent of this number. How many zeros
are there in M?
15. If N = 192837465564738291, then find the
(a) 90 (b) 89
remainder when (N + 1)(N + 2)… (N + 7) is
(c) 88 (d) 87
divided by 11.
(a) 7 (b) 5
20. A positive whole number M less than 100 is
(c) 2 (d) 9
represented in base 2 notation, base 3
notation and base 5 notation. It is found that
16. What is the remainder when 726 × 583 is
in all three cases the last digit is 1, while in
divided by 100?
exactly two out of the three cases the leading
(a) 50 (b) 25
digit is 1. Which of the following may be the
(c) 35 (d) 45
value of M?
(a) 31 (b) 63
17. 66n – 56n, where ‘n’ is an integer > 0, is
(c) 75 (d) 91
divisible by
(a) 11 (b) 91
(c) 341 (d) (a), (b) and (c)

Page 4 Number System


CEX–5670/20 QA – 01 : BBA + IPM Specific BBA+IPM

Answers and Explanations


Class Exercise
1 d 2 d 3 d 4 d 5 a 6 a 7 c 8 a 9 a 10 d
11 a 12 c 13 b 14 d 15 c 16 c 17 d 18 d 19 d 20 d
21 c 22 d 23 d 24 b 25 d

Home Exercise
1 d 2 d 3 b 4 c 5 a 6 d 7 d 8 c 9 a 10 d
11 d 12 a 13 d 14 a 15 c 16 b 17 d 18 b 19 c 20 d

Class Exercise 3. d Check the options:


Option (a), when x = 2, y = 3 and z = 5, then
1. d Let the two digit number be ‘ab’, then x+y 2+3
= = 1 . So, it can be true.
10a + b = a2 + b2 z 5
⇒ 10a – a2 = b2 – b Option (b), when x = 3, y = 13 and z = 17, then
⇒ a(10 – a) = b(b – 1) y+z 13 + 17
x2y + = 9 × 13 + = 117 + 10 = 127
Now, b(b – 1) is always even as the product of two x 3
consecutive digits is also even. So, it can be true.
⇒ a(10 – a) can be 2 × 8 = 16 or 4 × 6 = 24. Option (c), when x = 3, y = 5 and z = 7, then
Neither 16 nor 24 can be the product of two
x3 + 3y 27 + 15
consecutive numbers such as (b – 1) and b. = =6.
z 7
Hence, option (d) is the correct answer.
So, it can be true.
2. d Let, x = 2.52145145145... Hence, option (d) is the correct choice.
Or, 100x = 252.145145145... ...(i)
Also, 100000x = 252145.145145145... ...(ii) 4. d For a number to be divisible by 99, it must be divisible
Subtracting (ii) from (i) we get that by 9 as well as 11.
⇒ 99900x = (252145.145145145...) So, check divisibility for 9 and 11.
– (252.145145145...) Correct choice is option (d).
⇒ 99900x = 251893
5. a I. LCM (4, 5) = 20
251893 ∴ Numbers divisible by both 4 and 5 must be divisible
Therefore, x =
99900 by 20. Hence, numbers will be 220, 240, … 380, i.e.
9 numbers.
Alternative method: (Using the formula)
II. Here we just need to check that how many numbers
p are there between 200 and 400 that are divisible by 4
The form of any recurring number = [(the non –
q or 5, because that will take care of all the numbers in
recurring and recurring part written once) – (the non the range that are divisible by 8 or 10.
recurring part] / [as many 9’s as the number of digits in Total numbers divisible by 4 in the given range are 49.
recurring part followed by as many 0’s as the digits in Total numbers divisible by 5 in the given range are 39.
the non – recurring part after the decimal] Total numbers divisible by both 4 and 5 in the given
(252145 − 252) = 251893 range are 9.
=
(99900 ) 99900 ∴ The required numbers are 49 + 39 – 9 = 79.

Number System Page 1


6. a According to the given condition, The remainder when 117 is divided by 5 is 1.
10a + b = 6 (a + b) ⇒ The digit at the tens place of the given number N
⇒ 10a + b = 6a + 6b will be same as the digit at tens place of 66 which is 5.
⇒ 4a = 5b
13. b Two-digit numbers representing the year (yy), should
a 5 have the maximum number of factors so that we can
∴ =
b 4 express ‘yy’ as a product of two numbers (‘dd’ and
This implies that ratio of tens place digit to units place ‘mm’), maximum number of times.
digit is 5 : 4. Therefore, the number has to be 54. But we have constrains that ' dd' ≤ 31 and 'mm' ≤ 12.
Hence, ab = 54 = 625. One would observe that 60, 72, 84, 90 and 96 are
Note: Though two unknowns a and b are there, still those two digit numbers which have a maximum
one equation is sufficient to answer the equation as number of factors, i.e. 12.
‘a’ and ‘b’ are single digits. Out of these numbers, 60 has 6 possible cases for
(dd, mm) which are (20, 3), (15, 4), (5, 12), (12, 5),
7. c Let us consider a three-digit number ‘abc’. (10, 6) and (6, 10) which is the maximum. In all of
M = 100a + 10b + c – a – b – c = 99a + 9b these cases, ‘yy’ = 60.
Therefore, M is divisible by 9.
To get the largest possible 3-digit number put 14. d 180 = 22 × 32 × 51
a = 9, b = 9 Now, we should try to break-up 40 as a product of 3
∴ M = 99 × 9 + 9 × 9 = 972 factors.
Hence, N = 972 + 18 = 990. The only possible value of A satisfying all the given
conditions is 24 × 33 × 5
8. a It is given that ‘pqrs’ is four-digit number and ‘pq’ and
‘rs’ are both two-digit perfect squares. A 24 × 33 × 5
Hence, value of = = 54.
Two-digit perfect squares are 16, 25, 36, 49, 64, 81. 40 40
So, for every value of ‘pq’ there are five possible 15. c A33,6 represents set of natural numbers less than
values of ‘rs’. 10000 whose sum of digits is 33 and which are divisible
∴ Number of such four-digit numbers = 6 × 5 = 30. by 6. Clearly no element of A33,6 can be a number with
less than four digits as then sum of digits can never
9. a LCM of cyclicities of 1, 2, 3, … 9 is 4. be more than 27.
∴ If A is a single digit, then last digit of A4n+1 will be A
only. Thus we need to find number of four-digit numbers
Therefore, last digits are 1, 2, 3, 4 … 9 respectively. which have sum of digit as 33 and are divisible by 6.
Hence, last digit of the sum is 5. Divisibility rule for 6 states that the number should be
divisible by both 3 (which is true as sum is given to be
10. d The last two digits of any number is the remainder 33) and 2 (which indicates that the unit digit must be
when the number is divided by 100. even or zero).
29933 = (300 – 1)33.
So, the remainder when 29933 is divided by 100 is – 1. Also maximum possible sum of digits for any four-digit
Actual remainder is 100 – 1 = 99. number is 9 × 4 = 36.
As 33 is very close to 36 we can simply count those
numbers that are divisible by 6.
11. a (101) 1
= 101
Maximum possible sum of first three digits = 3 × 9 = 27.
(101)2 = 10201
Hence unit digit cannot be 0, 2 or 4 as then the sum
(101)3 = 1030301 would never reach 27.

Therefore, digit at the hundred’s place of the number Case 1: Unit digit is 6:
Only possible number is 9996.
n
N = (101) will always be ‘n’, for 0 < n ≤ 9 Case 2: Unit digit is 8:
Sum of first three digits must be 33 – 8 = 25.
9 This can happen in case where we have:
So, the digit at the hundreds place of (101) is 9.
3!
(i) Two 9’s and one 7. Which will give = 3 numbers.
12. c The digits at the tens place of 62, 63, 64, 65 and 2!
66 are 3, 1, 9, 7 and 5 respectively. (ii) Two 8’s and one 9. Which will give
Also, the digit at the ten’s place of 67 is 3. 3!
= 3 numbers.
⇒ The cyclicity of the tens digit of 6N is 5 (N ≥ 2). 2!
Answer = 1 + 3 + 3 = 7 possible numbers.

Page 2 Number System


16. c Strings vibrate 3, 4, 5, 6, 10, and 12 time in a min, i.e. 60
sec. Therefore time intervals between two vibrations B = 510 (1 × 2 × 3 × ...............10)

for each string are


60 60 60 60 60 60
, i.e. 20, B = 510 × 10!
, , , , ,
3 4 5 6 10 12 Highest power of 2 in 10! = 5 + 2 + 1 = 8
15, 12, 10, 6, 5 sec. Highest power of 5 in 10! = 2.
∴ Required time = LCM (20, 15, 12, 10, 6, 5)
B = 512 × 28 × M , where M is not a multiple of 2 or 5.
= 60 sec = 1 min.
∴ After 60 sec, all six strings will vibrate together. A
Therefore, in 30 mins, they will toll together 30 times. = 2189 × 512 × P, where P is not a multiple of 2 or 5.
B

17. d One of the numbers is 8! = 5! × (6.7.8). A


Let the other number be 5! × a. Highest power of 10 in = 12.
B
Hence, a should be relatively prime with (6.7.8) so
that the HCF remains 5! A
Number of zeros at the end of = 12.
In that case, LCM = a × 8! B
9! – 1 × 8! = 8 × 8!
9! – 2 × 8! = 7 × 8!
20. d Suppose x = 6k1 + 4 and y = 6k2 + 5
9! – 3 × 8! = 6 × 8!
x2 + y2 = (6k1 + 4)2 + (6k2 + 5)2
9! – 4 × 8! = 5 × 8!
= 36k12 + 48k1 + 16 + 36k22 + 60k2 + 25
If a = 5, it is co-prime with (6.7.8).
= 36k12 + 48k1 + 36k22 + 60k2 + 41
Hence, option (d) is a valid LCM.
Obviously, when this is divided by 6, the remainder
will be 5.
18. d Here, 2100! = (504)p × q
⇒ 2100! = 7p × 72p × q
Short cut:
Given q is not a multiple of 7.
So, number of 7’s in 2100! is the sum of r11 + r2 2 will also give the same remainder when divided
by 6 (where ‘r1’ and ‘r2’ are the remainders when ‘x’
2100 300 42
= 300 , = 42 and =6 and ‘y’ are divided by 6 respectively).
7 7 7
i.e. p = 300 + 42 + 6 = 348. 42 + 52 41
∴ = will give 5 as a remainder.
Hence, the value of p is 348. 6 6

21. c 784 = (74)21 = 240121 = (2402 – 1)21


19. d A = 2 × 4 × 6 × 8 × 10 × ................198 × 200
When (2402 – 1)21 is divided by 2402, the remainder
A = 2100 (1× 2 × 3 × 4 × 5...................100) will be 2401.
[Whenever (n – 1)k is divided by ‘n’, the remainder is
A = 2100 (100!) (n – 1), if k is odd].
Highest power of 2 in 100! = Here, k = 21, n = 2402.

100  100  100  100  100  100  22. d Any number in the form of an – bn is always divisible
 2 + 2 + 3 + 4 + 5 + 6 
  2  2  2  2  2  by (a – b).
= 50 + 25 + 12 + 6 + 3 + 1 = 97. Therefore, (7413 – 4113) and (7513 – 4213) is divisible
Similarly, highest power of 5 in 100! by 33.
(7413 – 4113) is odd and (7513 – 4213) is also odd.
100  100  So, (7413 – 4113 + 7513 – 4213) is even and hence
= +  = 20 + 4 =24.
 5   52  divisible by 2.
And we also know that (7413 – 4113 + 7513 – 4213) is
A = 2197 × 524 × N, where N is neither a multiple of 2
divisible by 33.
nor 5.
Therefore, (7413 – 4113 + 7513 – 4213) is also divisible
B = 5 × 10 × 15 × 20 × 25 × 30............. × 50 by 66. Hence, the remainder is zero.

Number System Page 3


23. d 51 = 3 × 17. Home Exercise
Let us first find the remainder when 531111 is divided
by 3. 1. d For every integer a, a + (–a) = 0.
531111 = (54 – 1)1111. So the remainder when 531111 is Therefore, by pairing 1 with –1, 2 with –2, and so on,
one can see that in order to get the sum to be zero, a
divided by 3 is when (– 1)1111 is divided by 3. list of consecutive integers must contain the same
Therefore, the remainder is 2. number of positive integers as the number of negative
So, 531111 can be written as 3k + 2, where ‘k’ is a integers. In addition to that it should also contain the
natural number. integer ‘0’.
Therefore, the list has an odd number of consecutive
Let us now find the remainder when 531111 is divided integers and their average will also be 0.
by 17. So, II and III are definitely true.
531111 = (51 + 2)1111. So the remainder is when 21111 is
divided by 17. 2. d Square of 625 ends in “0625”.
Square of 376 ends in “1376”.
Hence the required value of a + b + c + d cannot be
 2 . (2 ) ( )
 277 
( )
3 4 determined uniquely.
8. (17 − 1)
277
1111 3 1108
2 .2 
= =
2
=
17 17 17 17 3. b Whole numbers start from zero.
∴ First 100 whole numbers would be 0, 1, 2, …, 99.
Therefore, the remainder is 9. The occurrence of 0 in first 100 whole numbers will
So, 531111 can be written as 17m + 9, where ‘m’ is a be the following:
0, 10, 20, 30, 40, 50, 60, 70, 80, 90.
natural number.
3 Shaurya writes ‘0’ 10 times, A = 10.
Therefore, 17m + 9 = 3k + 2 The occurrence of ‘9’ in first 100 whole numbers will
Or, 17m = 3k – 7, therefore we can conclude that be in the following:
(m – 1) is a multiple of 3. 9, 19, 29, 39, 49, 59, 69, 79, 89, 90, 91, 92, 93, 94, 95,
531111 = 17m + 9 = 17(3y + 1) + 9, where ‘y’ is a natural 96, 97, 98, 99
3 Shaurya writes ‘9’ 20 times, B = 20.
number. Required sum = 10 + 20 = 30
531111 = 51y + 26.
Therefore, the remainder when 531111 is divided by 51 4. c Here ‘a’ is divisible by 7. From 125 to 275, there are 22
is 26. multiples of 7 out of which 11 numbers are even and
11 numbers are odd. Also, there are 2 multiples of 77
out of which one is odd and one is even.
24. b (12323)4 → ( )2 → (P)8 So, from 125 to 175, number of multiples of 7 but not of
11 is 11 – 1 = 10.
(12323 )4 → 110111011 

1 2 3 2 3  2
5. a Product of any 100 consecutive integers is always
Divide 110111011 into groups of 3 digits from right to
divisible by 1, 2, 3, 4, 5,.....98, 99 and 100. Thus, the
left. required remainder is 0.
(110 111 011)2 → (6 7 3 )8
6. d If N = ap × bq,
sets of factors which are coprime to each other can
25. d Consider the subtraction 54321 – 12345. For the unit
be directly calculated by the formula
digit, one unit, i.e. 6 (as the base is 6) will be carried
[(p + 1) (q + 1) – 1] + p × q
over from tens place and thus unit digit will be 1 + 6 –
Here a and b are prime factors and p and q are power
5 = 2. Similarly, tens place will be 1 + 6 – 4 = 3. Hundreds
of prime factors.
place will be 2 + 6 – 3 = 5. For the thousands place we If total n factors are formed, then there will be
do not need any carry over, hence it will be 3 – 2 = 1. (n – 1) pairs. Total factors have been subtracted by 1.
The left most digit will be 5 – 1 = 4. Thus, the subtraction Addition of p × q occurs as all of these factors will
will result in 41532. form a coprime with 1 also.
The expression is equal to In total, 17 cases are possible –
(4 × 6 4
) (
+ 1× 63 + 5 × 62 + 3 × 61 + 2 × 60 × 1× 62 ) (1, 2), (1, 3), (1, 4), (1, 6), (1, 8), (1, 9), (1, 12),
(1, 18), (1, 24), (1, 36), (1, 72) (2, 3), (2, 9), (4, 3),
= 4 × 66 + 1× 65 + 5 × 64 + 3 × 63 + 2 × 62 = 201600. (4, 9), (8, 3) and (8, 9).

Page 4 Number System


7. d N2 = 234 × 318 × 56. 12. a To check the highest power of 12 in 49!, we need to
Total number of factors of the number check the highest powers of 4 and 3 in it.
N2 = 35 × 19 × 7 = 4655. Highest power of 3 in 49! = 22
Highest power of 2 in 49! = 46
(4655 – 1)
There will be = 2327 pairs of factors of 46
2 ∴ Highest power of 4 in 49! = = 23
2
N2 such that the product of the factors is equal to N2. ∴ Highest power of 12 will be 22 (since the common
Apart from these, N will form a pair with N such that power between 3 and 4 is 22).
the product will give N2.
So, we can say that there will be 2327 factors of N2, 13. d The number of zeroes will be given by the highest
which are less than N. power of 5 in the number.
Total number of factors of N = 18 × 10 × 4 = 720. Now 2! and 4! do not contribute any power of 5
Each 6! contributes 1 power of 5
So 720 – 1 = 719 factors of N, which are less than N
∴ 6!6! contributes 6! powers of 5
and all of them are also factors of N2. Each 8! contributes 1 power of 5
Therefore, the number of divisors of N2 which are ∴ 8!8 contributes 8! powers of 5
less than N and do not divide N = 2327 – 719 = 1608. Each 10! contributes 2 powers of 5

∴ 10!10! contributes 2 × 10! powers of 5


8. c For 4 factors the number should be in the form of a1 ×
b1 or a3. (a and b are prime numbers) Thus, total number of zeroes = 6! + 8! + 2 × 10!
Case 1: if the number is in the form a1 × b1:
14. a To find the remainder when a number N is divided by
If a = 2, b can be any prime number up to 50 except b
3, we need to find the remainder when the sum of the
= 2 and 3. So there are 13 possibilities. digits of N is divided by 3. Since the sum of the digits of
If a = 3, b can be any prime number up to 33 except b A is 13, the number A when divided by 3 leaves a
= 2 and 3. So there are 9 possibilities. remainder of 1.
If a = 5, b can be any prime number up to 20 except b Hence, A4 = (3x + 1)4 leaves a remainder 1 when
= 2, 3 and 5. So there are 5 possibilities. divided by 3.
If a = 7, b can be 11 or 13. So there are 2 possiblities.
15. c Sum of the digits at the even places of the number N is
Case 2: if the number is in the form a3. The only possi-
equal to 9 + 8 + 7 + 6 + 5 + 4 + 3 + 2 + 1 = 45.
bility is 33 = 27
Sum of the digits at the odd places of the number N is
So total number of possibilities is 13 + 9 + 5 + 2 + 1 = 30 equal to 1 + 2 + 3 + 4 + 5 + 6 + 7 + 8 + 9 = 45.
Difference = 45 – 45 = 0.
9. a Since P is a prime number, so P and (P + 1) are coprime Therefore, the number N is divisible by 11.
∴ HCF of P and (P + 1) = 1 Hence, the remainder when (N + 1)(N + 2)…..
(N + 6)(N + 7) is divided by 11 is the remainder when
Their product P(P + 1) 1 × 2 × 3 × 4 × 5 × 6 × 7 is divided by 11, i.e. 2.
∴ LCM = = = P(P + 1).
HCF 1

10. d 120 = 23 × 3 × 5 726 × 583 726 × 583 726 × 581 (8 – 1)26 × (4 + 1)81
16. b = 2 = =
100 5 × 22 4 4
Number of elements in S satisfying the mentioned
Since (8 – 1)26 × (4 + 1)81 or 726 × 581 gives a remainder
 1  1  1 of 1 when divided by 4, then 726 × 583 gives a remainder
criteria = 120  1 −  1 −  1 −  = 32.
 2  3  5  of 1 × 25 when divided by 100
Therefore, the actual remainder is 25.
11. d Difference between the divisors and remainders is
constant, i.e. 2. 17. d an – bn (n > 0 and integer) is always divisible by
(a – b).
Therefore least number = LCM of divisors – Difference
∴ 66n – 56n = (62)3n – (52)3n = 363n – 253n
= LCM (3, 4, 5, 6, 7) – 2 = 420 – 2 = 418
∴ This is divisible by (36 – 25) = 11
Second number = LCM × 2 – Difference Similarly, 66n – 56n = (63)2n – (53)2n = 2162n – 1252n
= 420 × 2 – 2 = 838 ∴ This is divisible by (216 – 125) = 91 and
Third number = 420 × 3 – 2 (216 + 125) = 341.
= 1260 – 2 = 1258, which is not a three – digit number. 341 is divisible by 11, 91 and 341.
Therefore, there are 2 three-digit numbers. Hence, option (d) is the correct choice.

Number System Page 5


18. b 650 = (63)16 × 62 = (216)16 × 36 = (215 + 1)16 × 36 20. d Since the last digit in base 2, 3 and 5 is 1, the number
When this is divided by 215, remainder will be should be such that on dividing by either 2, 3 or 5 we
(1)16 × 36 = 36. should get a remainder 1. The smallest such number is
31. The next set of numbers are 61, 91.
19. c 389 in base 3 will be written as ‘one followed by 89 Among these only 31 and 91 are a part of the answer
choices.
zeroes’. When we add one to it, zero in unit’s place
will become one and we will have 88 zeros. Among these, (31)10 = (11111)2 = (1011)3 = (111)5
Thus, all three forms have leading digit 1.
Hence, the answer is 91.

Page 6 Number System

You might also like